1 Introduction and preliminaries

The study of inequalities with power-exponential functions is one of the active areas of research in the mathematical analysis. The power-exponential functions have useful applications in mathematical analysis and in other theories like statistics, biology, optimization, ordinary differential equations, and probability [1]. We note that the formulas of inequalities with power-exponential functions look so simple, but their solutions are not as simple as it seems. A lot of interesting results for inequalities with power-exponential functions have been obtained. The history and the literature review of inequalities with power-exponential functions can be found, for example, in [1]. Some other interesting problems concerning inequalities of power-exponential functions can be found in [2]. In this paper, we are studying one inequality conjectured by Cîrtoaje [3]. Cîrtoaje [3] has posted the following conjecture on the inequalities with power-exponential functions.

Conjecture 1.1

If \(a,b \in(0; 1]\) and \(r \in[0; e]\), then

$$ 2\sqrt{a^{ra}b^{rb}}\geq a^{rb}+b^{ra}. $$
(1.1)

The conjecture was proved by Matejíčka [4]. Matejíčka [5] also proved (1.1) under other conditions. Now we prove that the conjecture (1.1) is also valid under the following conditions:

  • \(\frac{2}{e}\leq\min\{a,b\}\leq1\) and \(1\leq\max\{a,b\}\leq e\) for \(r \in[0; e]\);

  • \(1\leq\min\{a,b\}\leq\max\{a,b\}\leq e\) for \(r \in[0; e]\).

We also show that a certain generalization of Cîrtoaje’s inequality fulfils an interesting property with some applications.

2 Main results

Theorem 2.1

Let \(a,b\) be positive numbers. Then

$$ 2\sqrt{a^{ra}b^{rb}}\geq a^{rb}+b^{ra} $$
(2.1)

for any \(r\in[0,e]\) if one of the following two conditions is satisfied:

$$\begin{aligned}& \frac{2}{e}\leq b\leq1\leq a\leq e; \end{aligned}$$
(2.2)
$$\begin{aligned}& 1\leq b\leq a\leq e. \end{aligned}$$
(2.3)

Proof

According to the proof of the Theorem 2.1 in [5], it suffices to consider the case where \(r=e\).

We split the proof into two parts, labeled as (a) and (b) with valid (2.2) and (2.3), respectively.

(a) Let a and b satisfy (2.2). Denote

$$ H(x)=2\sqrt{x^{ex}b^{eb}}-x^{eb}-b^{ex} $$

for \(x \in[1,e]\). We have

$$ H'(x)=e\bigl(x^{\frac{ex}{2}}b^{\frac{eb}{2}}(\ln x+1)-bx^{eb-1}-b^{ex}\ln b\bigr)=eb^{ex}F(x), $$

where

$$ F(x)=e^{\frac{e}{2} (x\ln x+b\ln b-2x\ln b )}(1+\ln x)-be^{(eb-1)\ln x-ex\ln b}-\ln b $$

and

$$\begin{aligned} F'(x) =& e^{\frac{e}{2} (x\ln x+b\ln b-2x\ln b )} \biggl(\frac {e}{2} (1+\ln x ) (1+ \ln x-2\ln b )+\frac{1}{x} \biggr) \\ &{} -be^{(eb-1)\ln x-ex\ln b} \biggl(\frac{eb-1}{x}-e\ln b \biggr). \end{aligned}$$

If we show that \(H(1)\geq0\) and \(H'(x)\geq0\) for \(x \in[1,e]\), then the proof will be done.

To prove that \(H(1)\geq0\), we consider the function \(s: [2/e,1] \rightarrow\mathbf{R}\) defined as

$$ s(b)=H(1)=2b^{\frac{eb}{2}}-1-b^{e}. $$

We have that \(s(1)=0\). Now, if we show that \(s'(b)\leq0\) for \(b\in [2/e,1]\), then we can conclude that \(H(1)\geq0\). From

$$ s'(b)=e^{\frac{eb}{2}\ln b+1}(\ln b+1)-eb^{e-1} $$

we obtain that \(s'(b)\leq0\) is equivalent to

$$ \frac{eb\ln b}{2}-(e-1)\ln b+\ln(1+\ln b)\leq0 \quad\mbox{for } \frac{2}{e}\leq b\leq1. $$

Using

$$ \ln(1+\ln b)\leq\ln b-\frac{\ln^{2}b}{2}, $$

it suffices to show that

$$ v(b)=\frac{eb}{2}+2-e-\frac{\ln b}{2}\geq0. $$

But the latter follows from \(v(2/e)=3-e+\frac{1}{2}\ln (\frac {e}{2} )>0\) and from \(v'(b)=(eb-1)/(2b)>0\).

Now we show \(F'(x)\geq0\) for \(x\in[1,e]\). This implies \(H'(x)\geq0\) for \(x\in[1,e]\). Indeed, if we show that \(F(1)\geq0\) and \(F'(x)\geq 0\), then \(F(x)\geq0\), so that \(H'(x)\geq0\).

We have

$$ F(1)=e^{\frac{eb}{2}\ln b-e\ln b}-be^{-e\ln b}-\ln b. $$

Because of \((\frac{eb}{2}-e)<-\frac{e}{2}\), it suffices to show that

$$ y(b)=b^{-\frac{e}{2}}-b^{1-e}-\ln b\geq0 $$

for \(\frac{2}{e}\leq b\leq1\).

Since \(y(1)=0\), it suffices to show that

$$ y'=-\frac{e}{2}b^{-\frac{e}{2}-1}-(1-e)b^{-e}- \frac{1}{b}\leq0. $$

This is equivalent to

$$ g=-\frac{e}{2}b^{-\frac{e}{2}}+(e-1)b^{1-e}\leq1, $$

which follows from \(g(2/e)=0.8488\) and from

$$ g'=\frac{e^{2}}{4}b^{-\frac{e}{2}-1}-(e-1)^{2}b^{-e} \leq0. $$

Indeed, \(g'<0\) follows from \(b\leq1< e/2\).

Next, we have that \(F'(x)\geq0\) is equivalent to

$$ e^{\frac{e}{2} (x\ln x+b\ln b-2x\ln b )}\geq \frac{be^{(eb-1)\ln x-ex\ln b}(eb-1-ex\ln b)}{ \frac{ex}{2}(\ln x+1)(\ln x+1-2\ln b)+1}. $$
(2.4)

This can be rewritten as

$$\begin{aligned} &{\frac{e}{2} (x\ln x+b\ln b )}-(eb-1)\ln x \\ & \quad\geq\ln \bigl(2b(eb-1-ex\ln b) \bigr)-\ln \bigl(ex(\ln x+1) (\ln x+1-2\ln b)+2 \bigr). \end{aligned}$$
(2.5)

Evidently,

$$ ex(\ln x+1) (\ln x+1-2\ln b)+2\geq ex+2. $$

So, to prove \(F'(x)\geq0\), it suffices to show that

$$\begin{aligned} &{\frac{e}{2} (x\ln x+b\ln b )}-(eb-1)\ln x \\ &\quad \geq\ln \bigl(2b(eb-1-ex\ln b) \bigr)-\ln(ex+2). \end{aligned}$$

Using \(\ln b>(b-1)/b\), we obtain

$$ 2b(eb-1-ex\ln b)< 2eb^{2}-2b+2ex(1-b). $$

So we need to show that

$$\begin{aligned} &{\frac{e}{2} (x\ln x+b\ln b )}-(eb-1)\ln x \\ &\quad \geq\ln \bigl(2eb^{2}-2b+2ex(1-b) \bigr)-\ln(ex+2). \end{aligned}$$

Using again \(\ln b>(b-1)/b\) and \(\ln x>(x-1)/x\), it suffices to show that

$$\begin{aligned} r(x)={}&\frac{e}{2} (x+b-2 )-(eb-1)\ln x \\ &{} -\ln \bigl(2eb^{2}-2b+2ex(1-b) \bigr)+\ln(ex+2)\geq0. \end{aligned}$$

Because of \(\ln x< x-1\), it suffices to prove that

$$\begin{aligned} r^{\ast}(x)={}&\frac{e}{2} (x+b-2 )-(eb-1) (x-1) \\ &{}-\ln \bigl(2eb^{2}-2b+2ex(1-b) \bigr)+\ln(ex+2)\geq0. \end{aligned}$$

It will be done if we show that \(r^{\ast\prime\prime}(x)\leq0\), \(r^{\ast }(1)\geq0\), and \(r^{\ast}(e)\geq0\).

We have

$$ r^{\ast\prime\prime}(x)= \frac{4e^{2}(1-b)^{2}}{(2eb^{2}-2b+2ex(1-b))^{2}}-\frac {e^{2}}{(ex+2)^{2}}. $$

Because of \(r^{\ast\prime\prime}(x)=0\) only for one real root \(x_{1}=(eb^{2}-3b+2)/(2be-2e)<0\), we obtain \(r^{\ast\prime\prime}(x)\leq0\) for \(2/e\leq b<1\) and \(1\leq x\leq e\).

Now we show that \(r^{\ast}(e)\geq0\). We have

$$\begin{aligned} r^{\ast}(e)&= u(b) \\ &= \frac{e^{2}}{2}-1+b \biggl(\frac{3}{2}e-e^{2} \biggr)-\ln \bigl(2e^{2}-2b\bigl(1+e^{2} \bigr)+2eb^{2} \bigr)+\ln\bigl(e^{2}+2\bigr)\geq0. \end{aligned}$$

First, we show that \(u(1)\geq0\) and then \(u'(b)<0\). We have

$$ u(1)=\frac{e^{2}}{2}-1+\frac{3}{2}e-e^{2}-\ln (2e-2 )+\ln \bigl(e^{2}+2\bigr)\doteq0.388>0. $$

Since

$$ u'(b)=\frac{3e}{2}-e^{2}-\frac{4eb-2-2e^{2}}{2e^{2}-2b(1+e^{2})+2eb^{2}}, $$

we obtain that \(u'(b)<0\) is equivalent to

$$ k(b)=2+2e^{2}+3e^{3}-2e^{4}-2b \bigl(1+e^{2}\bigr) \biggl(\frac{3e}{2}-e^{2} \biggr)-4eb+2e \biggl(\frac{3e}{2}-e^{2} \biggr)b^{2} \leq0. $$

It is evident that \(k(b)\) is a concave function. We show that \(k'=0\) only for \(m>1\) and \(k(1)<0\). This implies that \(k(b)<0\) for \(2/e\leq b\leq1\). So \(u'(b)<0\). Indeed, if \(k'=0\), then

$$ m=\frac{(1+e^{2})(2e^{2}-3e)-4e}{2e(2e^{2}-3e)}=\frac {2e^{3}-3e^{2}+2e-7}{4e^{2}-6e}\doteq1.2411. $$

We also have

$$ k(1)=2+2e^{2}+3e^{3}-2e^{4}-2b \bigl(1+e^{2}\bigr) \biggl(3\frac{e}{2}-e^{2} \biggr)-4eb+2e\biggl(3\frac {e}{2}-e^{2}\biggr)b^{2} \doteq-5.4757< 0 . $$

Now we show that \(r^{\ast}(1)\geq0\). It will be done if we prove

$$ t(b)=\frac{e}{2}(b-1)-\ln\bigl(2eb^{2}-2b+2e-2eb\bigr)+ \ln(e+2)\geq0. $$

But this follows from \(t'(b)\geq0\) and \(t(2/e)\geq0\). We have

$$ t \biggl(\frac{2}{e} \biggr)=1-\frac{e}{2}-\ln \biggl( \frac{4}{e}+2e-4 \biggr)+\ln(e+2)\doteq0.1248\geq0 $$

and

$$ t'(b)=\frac{e}{2}-\frac{4eb-2e-2}{2eb^{2}-2b+2e-2eb}. $$

The inequality \(t'(b)\geq0\) is equivalent to

$$ n(b)=e^{2}b^{2}-b\bigl(5e+e^{2} \bigr)+e^{2}+2e+2\geq0 $$

since \(o(b)=2eb^{2}-2b+2e-2eb\geq0\), which is evident (\(o''(b)>0\), \(o'(b)=0\) for \(b=(1+e)/(2e)<2/e, o(2/e)>0\)). Now \(n(b)\geq0\) follows from \(n''(b)\geq0\), \(n'(b)=0\) for \(b=(5+e)/(2e)>1\), and \(n(1)=e^{2}-3e+2\doteq1.2342\geq0\).

(b) We assume that a and b satisfy (2.3).

We show again that \(H'(x)\geq0\) but now for \(1\leq b\leq x\leq e\). Because of \(H(b)=0\), the proof will be done.

From (2.4) we have that if \((eb-1-ex\ln b)\leq0\), then \(F'(x)\geq 0\). So we need to show that \(F'(x)\geq0\) for \(s(x,b)=(eb-1-ex\ln b)>0\).

Let \(s(x,b)=(eb-1-ex\ln b)>0\) for \(1\leq b\leq x\leq e\). Then \(F'(x)\geq0\) if only if

$$\begin{aligned} f(x,b)={}&{\frac{e}{2} (x\ln x+b\ln b )}-(eb-1)\ln x-\ln \bigl(2b(eb-1-ex\ln b) \bigr) \\ &{} +\ln \bigl(ex(\ln x+1) (\ln x+1-2\ln b)+2 \bigr)\geq0. \end{aligned}$$
(2.6)

If \((eb-1-ex\ln b)>0\), then \(x<\frac{eb-1}{e\ln b}\). Because of \(x>b\), we have \(be\ln b< eb-1\). Put \(t=eb\) and \(v(t)=t\ln t-2t+1\) for \(e\leq t\leq e^{2}\). Then we have \(v(e)=1-e<0\), \(v(e^{2})=1\), \(v'(t)=\ln t-1>0\). This implies that there is only one \(t^{\ast}\) such that \(e< t^{\ast}< e^{2}\) and \(v(t^{\ast})=0\). Because of \(v(6.3055)=8.8113e-005>0\), we get \(t^{\ast}<6.3055\). So \(b< b^{\ast }<2.3196\). This implies that it suffices to show \(f(x,b)\geq0\) for \(1< b<2.3196\).

The mean value theorem gives

$$ \ln x-\ln b+1-\ln b >\frac{1}{x}(x-b)+\frac{1}{e}(e-b),\qquad \ln x+1\geq\frac{2x-1}{x}. $$

This implies

$$\begin{aligned} & \ln \bigl(ex(\ln x+1) (\ln x+1-2\ln b)+2 \bigr) \\ &\quad \geq\ln \bigl(e(2x-1) (2ex-eb-xb)+2ex \bigr)-\ln x-1. \end{aligned}$$

Similarly,

$$\begin{aligned} x\ln x+b\ln b-2b\ln x&=(x-b)\ln x+b(\ln b-\ln x)\geq(x-b) (\ln x-\ln e) \\ & >-\frac{(x-b)(e-x)}{x}. \end{aligned}$$

So

$$ \frac{e}{2} (x\ln x+b\ln b )-eb\ln x\geq-\frac{e}{2} \frac {(x-b)(e-x)}{x}. $$

From \(\ln b>\frac{2(b-1)}{b+1}\) we have \(f(x,b)\geq G(x,b)\), where

$$\begin{aligned} G(x,b)={}&{-}\frac{e}{2}\frac{(x-b)(e-x)}{x}-\ln \biggl(2b \biggl(eb-1- \frac {2ex(b-1)}{1+b} \biggr) \biggr) \\ &{}+\ln \bigl((2x-1) (2ex-eb-xb)+2x \bigr). \end{aligned}$$

We show that \(G(b,b)\geq0\) and \(G'_{x}(x,b)\geq0\), and the proof will be done.

We have

$$\begin{aligned}& G(b,b)=-\ln \biggl(2b \biggl(eb-1-\frac{2eb(b-1)}{1+b} \biggr) \biggr)+\ln \bigl((2b-1) \bigl(eb-b^{2}\bigr)+2b \bigr)= \\& L(b)=\ln \biggl(\frac{1+b}{2} \biggr)-\ln \bigl(-eb^{2}+3eb-b-1 \bigr)+\ln \bigl(-2b^{2}+2eb+b-e+2 \bigr). \end{aligned}$$

If we show that

$$ \frac{-2b^{2}+2eb+b-e+2}{-eb^{2}+3eb-b-1}\geq1, $$
(2.7)

then \(L(b)\geq0\), so \(G(b,b)\geq0\).

Inequality (2.7) is equivalent to

$$ s(b)=(e-2)b^{2}+(2-e)b+3-e\geq0. $$

From \(s'(b)=2(e-2)b+(2-e)\), \(s'(b)=0\) if \(b=0.5\), \(s(1)=3-e>0\) we have \(s(b)>0\), so \(G(b,b)\geq0\).

Now we show \(G'_{x}(x,b)\geq0\) for \(1< b< x<\min\{e,(eb-1)/(e\ln b)\}\) and \(1< b< b^{\ast}\).

Because of \(eb-1-\frac{2ex(b-1)}{1+b}>eb-1-ex\ln(b)>0\), we have

$$\begin{aligned} G'_{x}(x,b)\geq{}&{-}\frac{e}{2} \biggl( \frac{be-x^{2}}{x^{2}} \biggr)+\frac {2e(b-1)}{b ((eb-1)(1+b)-2ex(b-1) )} \\ &{} +\frac{8ex-4bx-2be-2e+b+2}{(2x-1)(2ex-eb-xb)+2x}. \end{aligned}$$

(We omitted a positive term of derivation \(G'_{x}(x,b)\).)

Since

$$ \frac{be-x^{2}}{x^{2}}< \frac{x(e-x)}{x^{2}}, \qquad\frac{2e(b-1)}{b ((eb-1)(1+b)-2ex(b-1) )}>0, $$

it suffices to show that

$$ \frac{xe-e^{2}}{2x}+\frac{8ex-4bx-2be-2e+b+2}{(2x-1)(2ex-eb-xb)+2x}\geq 0. $$
(2.8)

To prove (2.8), it suffices to show that (we used \(x>b\))

$$\begin{aligned} & x^{2}\bigl((4e-2b) \bigl(eb-e^{2}\bigr)+16e-8b\bigr) \\ &\quad{} +x\bigl(\bigl(eb-e^{2}\bigr) (2+b-2eb-2e)-4eb-4e+2b+4 \bigr)+eb\bigl(eb-e^{2}\bigr)\geq0. \end{aligned}$$

This can be rewritten as

$$ T(x,b)=x^{2}u(b)+xv(b)+w(b)\geq0, $$

where

$$\begin{aligned} & u(b)=-2eb^{2}+b\bigl(6e^{2}-8\bigr)+e \bigl(16-4e^{2}\bigr), \\ & v(b)=b^{2}\bigl(e-2e^{2}\bigr)+b\bigl(2-2e-3e^{2}+2e^{3} \bigr)+2e^{3}-2e^{2}-4e+4, \\ & w(b)=e^{2}b^{2}-be^{3}. \end{aligned}$$

From this we obtain that the roots of u are \(b_{1}=1.2468\) and \(b_{2}=5.4366\). We have that \(u<0\) on \((1,b_{1})\) and \(u>0\) on \((b_{1},b^{\ast})\). If we show that \(T(b,b)\geq0\), \(T(e,b)\geq0\) for \(b\in(1,b_{1})\) (\(T(x)\) is a concave function), and \(T'_{x}(x,b)\geq0\), \(T(b,b)\geq0\) for \(b\in(b_{1},b^{\ast})\), then the proof will be complete. Because of \(T'_{x}(x,b)=2xu+v\), it suffices to prove that \(P(b)=2bu(b)+v(b)\geq0\) for \(b\in(b_{1},b^{\ast})\).

First, we show \(T(b,b)\geq0\), \(T(e,b)\geq0\) for \(b\in(1,b_{1})\). We have

$$\begin{aligned} T(b,b)={}& b\bigl(-2eb^{3}+b^{2}\bigl(4e^{2}+e-8 \bigr)+b\bigl(2+14e-2e^{2}-2e^{3}\bigr) \\ &{}+4-4e-2e^{2}+e^{3}\bigr). \end{aligned}$$

The roots of \(T(b,b)=0\) are \(r_{1}=-0.1913\), \(r_{2}=0.8517\), \(r_{3}=3.7046\), \(r_{4}=0\). This implies that \(T(b,b)\geq0\) for \(b\in(1,e)\).

Next, we have

$$ T(e,b)=2e \bigl(b^{2}\bigl(-2e^{2}+e\bigr)+b \bigl(4e^{3}-2e^{2}-5e+1\bigr)-2e^{4}+e^{3}+7e^{2}-2e+2 \bigr). $$

The roots of \(T(e,b)=0\) are \(r_{1}=0.9969\), \(r_{2}=3.3956\), and \(T(e,0)<0\) implies \(T(e,b)\geq0\) for \(b\in(1,e)\).

Now we show that \(P(b)\geq0\) for \(b\in(b_{1},b^{\ast})\). We have

$$ P(b)=-4eb^{3}+b^{2}\bigl(10e^{2}+e-16\bigr)+b \bigl(-6e^{3}-3e^{2}+30e+2\bigr)+2e^{3}-2e^{2}-4e+4. $$

Because of \(P(b)=0\) has only one real root \(r_{1}=4.4344\), \(P(0)=18.5198\), and \(T(e,0)<0\), we obtain that \(p(b)\geq0\) for \(b\in (b_{1},b^{\ast})\).

So the proof is complete. □

3 Some generalizations of Conjecture 1.1

Denote \(M^{\ast}=\{(a,b); (0< a, b\leq e) \vee(0< b, a\leq e) \vee(a\geq e^{2}, b\leq\sqrt{a}) \vee(b\geq e^{2}, a\leq \sqrt{b}) \vee(0< a=b)\}\) (see Figure 1) and \(M(n,r)=\{(x_{1},\ldots,x_{n}); x_{i}>0, r\geq0 \wedge x_{1},\ldots,x_{n} \mbox{ are solutions of the}\mbox{ }\mbox{inequality (3.3)}\}\).

Figure 1
figure 1

Part of the set \(\pmb{M^{\ast}}\) .

We have:

  • \(\{(a,b); 0< a,b\leq e\}\subset M(2,e)\) (see [5]).

  • \(\forall s>e\), \(\exists a,b<1\) such that \((a,b)\not\subset M(2,s)\) (see [4]).

  • \(M^{\ast}\subset M(2,e)\).

  • \((5,10)\not\subset M(2,e)\) (see [5]).

  • \((1/3,1/9,2/3)\not\subset M(3,5/2)\) (see [5]).

  • If \(0< r< s\), then \(M(n,s)\subset M(n,r)\) (Note 3.4).

  • \(\forall x_{1},\ldots,x_{n}>0\), \(\exists s>0\) such that \((x_{1},\ldots,x_{n})\in M(n,r)\) for \(0\leq r\leq s\) (Note 3.4).

Lemma 3.1

[6], the log-sum inequality

Let \(n\in\mathbf {N}\), \(x_{1},\ldots,x_{n}, y_{1},\ldots,y_{n}\) be positive numbers. Then

$$ \sum_{i=1}^{n}x_{i}\ln \biggl( \frac{x_{i}}{y_{i}} \biggr)\geq \Biggl(\sum_{i=1}^{n}x_{i} \Biggr) \ln \biggl(\frac{\sum_{i=1}^{n}x_{i}}{\sum_{i=1}^{n}y_{i}} \biggr) $$
(3.1)

with equality only for \(\frac{x_{1}}{y_{1}}=\frac {x_{2}}{y_{2}}=\cdots=\frac{x_{n}}{y_{n}}\).

Lemma 3.2

Let

$$ F(r)=\ln n+\frac{r}{n} \Biggl(\sum_{i=1}^{n}x_{i} \ln x_{i} \Biggr)-\ln \Biggl(e^{rx_{1}\ln x_{n}} +\sum _{i=1}^{n-1}e^{rx_{i+1}\ln x_{i}} \Biggr), $$
(3.2)

where \(r\geq0\), \(n\in\mathbf{N}\), \(n\geq2\), \(x_{1},\ldots,x_{n}>0 \wedge \exists i\neq j\) such that \(x_{i}\neq x_{j}\). Then \(F(0)=0\), \(F'(0)>0\), \(F''(r)<0\).

Note 3.3

We note that \(F(r)\geq0\) is equivalent to

$$ n\sqrt[n]{\prod_{i=1}^{n}x_{i}^{rx_{i}}} \geq x_{n}^{rx_{1}}+\sum_{i=1}^{n-1}x_{i}^{rx_{i+1}}. $$
(3.3)

Proof

It is evident that \(F(0)=0\). Next, we have

$$ F'(r)=\frac{1}{n} \Biggl(\sum_{i=1}^{n}x_{i} \ln x_{i} \Biggr)-\frac {e^{rx_{1}\ln x_{n}}x_{1}\ln x_{n} +\sum_{i=1}^{n-1}e^{rx_{i+1}\ln x_{i}}x_{i+1}\ln x_{i}}{ e^{rx_{1}\ln x_{n}} +\sum_{i=1}^{n-1}e^{rx_{i+1}\ln x_{i}}}. $$

The inequality \(F'(0)>0\) is equivalent to

$$ \sum_{i=1}^{n}x_{i}\ln x_{i}-x_{1}\ln x_{n}-\sum _{i=1}^{n-1}x_{i+1}\ln x_{i}>0, $$

which can be rewritten as

$$ \sum_{i=2}^{n}x_{i}\ln \biggl( \frac{x_{i}}{x_{i-1}} \biggr)+x_{1}\ln \biggl(\frac{x_{1}}{x_{n}} \biggr)>0. $$
(3.4)

To prove (3.4), we use the Jensens log-sum inequality (Lemma 3.1).

Put \(y_{1}=x_{n}\), \(y_{2}=x_{1}, \dots,y_{n}=x_{n-1}\) in (3.1). We obtain

$$ \sum_{i=2}^{n}x_{i}\ln \biggl( \frac{x_{i}}{x_{i-1}} \biggr)+x_{1}\ln \biggl(\frac{x_{1}}{x_{n}} \biggr) \geq v= \Biggl(\sum_{i=2}^{n}x_{i} \Biggr) \ln \biggl(\frac{\sum_{i=2}^{n}x_{i}}{\sum_{i=1}^{n-1}x_{i}} \biggr)+x_{1}\ln \biggl( \frac{x_{1}}{x_{n}} \biggr). $$
(3.5)

We show that \(v=v(y,x_{1},x_{n})>0\), where \(y=\sum_{i=2}^{n-1}x_{i}\). We have

$$ v(y,x_{1},x_{n})= (y+x_{n} )\ln \biggl( \frac {y+x_{n}}{y+x_{1}} \biggr)+x_{1}\ln \biggl(\frac{x_{1}}{x_{n}} \biggr). $$

It is evident that \(v(0,x_{1},x_{n})=(x_{n}-x_{1})\ln (\frac {x_{n}}{x_{1}} )>0\) and

$$ v'_{y}(y,x_{1},x_{n})=\ln \biggl( \frac{y+x_{n}}{y+x_{1}} \biggr)+\frac {x_{1}-x_{n}}{y+x_{1}}. $$
(3.6)

If we show \(v'_{y}(y,x_{1},x_{n})\leq0\), then \(\lim_{y\rightarrow+\infty}v(y,x_{1},x_{n})=x_{n}-x_{1}+x_{1}\ln\frac {x_{1}}{x_{n}}\geq0\) (if we put \(t=x_{1}/x_{n}\), then \(g=1-t+t\ln t\geq 0\)) implies \(v(y,x_{1},x_{n})\geq0\).

Put \(t=\frac{y+x_{n}}{y+x_{1}}\) in (3.6). Then \(v'_{y}(y,x_{1},x_{n})=\ln t+1-t\). This implies \(v'_{y}(y,x_{1},x_{n})<0\).

Now we prove \(F''(r)<0\). We have

$$ F''(r)=\frac{-L(r)}{ (\exp (rx_{1}\ln x_{n} )+\sum_{i=1}^{n-1}\exp (rx_{i+1}\ln x_{i} ) )^{2}}< 0, $$

where

$$\begin{aligned} L(r)={}& \Biggl(\exp (rx_{1}\ln x_{n} )x_{1}^{2} \ln^{2} x_{n}+\sum_{i=1}^{n-1} \exp (rx_{i+1}\ln x_{i} )x_{i+1}^{2} \ln^{2} x_{i} \Biggr) \\ &{}\times \Biggl(\exp (rx_{1}\ln x_{n} )+\sum _{i=1}^{n-1}\exp (rx_{i+1}\ln x_{i} ) \Biggr) \\ &{}- \Biggl(\exp (rx_{1}\ln x_{n} )x_{1}\ln x_{n}+\sum_{i=1}^{n-1}\exp (rx_{i+1}\ln x_{i} )x_{i+1}\ln x_{i} \Biggr)^{2}\geq0. \end{aligned}$$

The equality \(L(r)\geq0\) can be rewritten as

$$\begin{aligned} L(r)={}&A_{n}+B_{n}=\sum_{i=1}^{n-1} \exp \bigl(r(x_{i+1}\ln x_{i}+x_{1}\ln x_{n}) \bigr) (x_{i+1}\ln x_{i}-x_{1} \ln x_{n})^{2} \\ &{}+\sum_{i=1}^{n-1}\sum _{j=1}^{n-1}\exp \bigl(r(x_{i+1}\ln x_{i}+x_{j+1}\ln x_{j} )\bigr) \bigl(x_{i+1}^{2} \ln^{2} x_{i}-x_{i+1}x_{j+1}(\ln x_{i})\ln x_{j} \bigr)\geq0. \end{aligned}$$

From \(B_{2}\geq0\) and

$$ B_{n+1}=B_{n}+\sum_{i=1}^{n-1} \exp \bigl(r(x_{i+1}\ln x_{i}+x_{n+1}\ln x_{n} )\bigr) (x_{i+1}\ln x_{i}-x_{n+1}\ln x_{n} )^{2} $$

we have \(A_{n}+B_{n}\geq0\) for \(n\geq2\). So, \(F(r)\) is a concave function for \(r\geq0\). □

Note 3.4

We note that Lemma  3.2 implies: if \(F(s)\geq0\) for some \(s>0\) and for positive numbers \(x_{1},\ldots,x_{n} \in M(n,s)\), then \(F(r)\geq0\) for \(r\in[0,s]\) on \(M(n,s)\).

3.1 Other applications of Lemma 3.2

  • For each \(A\in R^{n}_{+}=\{(x_{1},\ldots,x_{n}), x_{i}>0, i=1,\ldots,n\}\), \(n\in\mathbf{N}\), there is a finite limit \(L_{A}=\lim_{r\rightarrow+\infty}F'(r)=\frac{1}{n}\sum_{i=1}^{n}x_{i}\log(x_{i})-m_{x}\), where \(m_{x}=\max_{1\leq m\leq n}\{x_{m+1}\log(x_{m})\}\), \(x_{n+1}=x_{1}\).

  • Denote by \(r_{A}\) the positive root of \(F(r)=0\) (if the root exists) for \(A\in R^{n}_{+}\)-\(S^{n}\) where \(S^{n}=\{(x_{1},\ldots,x_{n}), x_{i}=x_{j}, i,j=1,\ldots,n\}\). Then

    1. (a)

      \(L_{A}\geq0\) ⇔ there is no \(r_{A}>0\) such that \(F(r_{A})=0\).

    2. (b)

      \(L_{A}<0\) ⇔ there is \(r_{A}>0\) such that \(F(r_{A})=0\).

Let \(\emptyset\neq M\subset R^{n}_{+}-S^{n}\). Put \(r_{M}=\inf_{A\in M}\{r_{A}\}\) and \(R_{M}=\sup_{A\in M}\{r_{A}\}\). Then there are seven cases:

  1. (a)

    \(r_{M}=R_{M}=0\),

  2. (b)

    \(0=r_{M}< R_{M}<\infty\),

  3. (c)

    \(r_{M}=0, R_{M}=\infty\),

  4. (d)

    \(0< r_{M}=R_{M}<\infty\),

  5. (e)

    \(0< r_{M}< R_{M}<\infty\),

  6. (f)

    \(0< r_{M}< R_{M}=\infty\),

  7. (g)

    \(r_{M}=R_{M}=\infty\).

From this we have:

  • Case (a) is not possible (Lemma 3.2).

  • In case (b), inequality (3.3) is not valid for \(r>0\) on M, but the reverse inequality to (3.3) is valid for \(r>R_{M}\) on M.

  • In case (c), inequality (3.3) and the reverse inequality to (3.3) are not valid for \(r>0\) on M.

  • In case (d), inequality (3.3) is valid for \(0\leq r\leq r_{M}\) on M, and the reverse inequality to (3.3) is valid for \(r>r_{M}\) on M.

  • In case (e), inequality (3.3) is valid for \(0\leq r\leq r_{M}\) on M, but the reverse inequality to (3.3) is valid for \(r>R_{M}>r_{M}\) on M.

  • In case (f), inequality (3.3) is valid for \(0\leq r\leq r_{M}<\infty\) on M, but the reverse inequality to (3.3) is not valid for any \(r>0\) on M.

  • In case (g), inequality (3.3) is valid for all \(r\geq0\) on M.

3.2 Example

Let \(n=2\). Denote \(a=x_{2}\), \(b=x_{1}\). Then (1.1) is equivalent to \(F(r)\geq0\).

We have three cases:

$$\left \{ \textstyle\begin{array}{@{}l@{\quad}l} b\log(a)>a\log(b); & \mbox{then } L_{A}=\lim_{r\rightarrow+\infty}F'(r)= (\frac{a-b}{2} )\log(a)+\frac {b}{2}\log (\frac{b}{a} );\\ a\log(b)>b\log(a); & \mbox{then } L_{A}=\lim_{r\rightarrow+\infty}F'(r)= (\frac{b-a}{2} )\log(b)+\frac {a}{2}\log (\frac{a}{b} );\\ b\log(a)=a\log(b); & \mbox{then } L_{A}=\lim_{r\rightarrow+\infty}F'(r)= (\frac{a-b}{2} )\log (\frac {a}{b} )\geq0. \end{array}\displaystyle \right . $$

Let

$$M=\bigl\{ (a,b); 0< b< a\leq1\bigr\} . $$

From \(b< a\) we have \(a\log(b)< b\log(a)\), so \((\frac{a-b}{2} )\log(a)+\frac{b}{2}\log (\frac{b}{a} )<0\). Lemma 2.2 in [4] gives that \(r_{M}=e\). \(\lim_{a\rightarrow1, b\rightarrow 0 }F(r)=\log2\) implies that \(R_{M}=\infty\). So, we have that the reverse inequality to (3.3) cannot be valid for any \(r>0\) on M.